Step 1: Understand the formula for spin-only magnetic moment.
The spin-only magnetic moment ($\mu_s$) is given by the formula:
$\mu_s = \sqrt{n(n+2)}\operatorname{BM}$ where 'n' is the number of unpaired electrons and BM stands for Bohr Magneton. We are given $\mu_s = 4.90\operatorname{BM}$.
Let's find the number of unpaired electrons (n) corresponding to this magnetic moment.
$4.90 = \sqrt{n(n+2)}$
Squaring both sides:
$(4.90)^2 = n(n+2)$
$24.01 = n^2 + 2n$
$n^2 + 2n - 24.01 = 0$
We can approximate $n$ to be 4 since $\sqrt{4(4+2)} = \sqrt{4 \times 6} = \sqrt{24} \approx 4.898$.
So, we are looking for a complex ion with 4 unpaired electrons.
Step 2: Determine the number of unpaired electrons for each complex ion option.
1. [Co(NH\(_3\))\(_6\)]\(^{3+}\)
Oxidation state of Co: Let it be $x$. $x + 6(0) = +3 \implies x = +3$.
Cobalt (Co) has atomic number 27. Electronic configuration: [Ar] 3d\(^7\) 4s\(^2\).
Co\(^{3+}\) electronic configuration: [Ar] 3d\(^6\).
NH\(_3\) (ammonia) is a strong field ligand.
In the presence of a strong field ligand, the 3d\(^6\) electrons pair up.
$t_{2g}^6 e_g^0$.
Number of unpaired electrons (n) = 0.
$\mu_s = \sqrt{0(0+2)} = 0\operatorname{BM}$. This is not 4.90 BM.
2. [Cr(NH\(_3\))\(_6\)]\(^{3+}\)
Oxidation state of Cr: Let it be $x$. $x + 6(0) = +3 \implies x = +3$.
Chromium (Cr) has atomic number 24. Electronic configuration: [Ar] 3d\(^5\) 4s\(^1\).
Cr\(^{3+}\) electronic configuration: [Ar] 3d\(^3\).
NH\(_3\) is a strong field ligand. However, for d\(^3\) configuration, irrespective of strong or weak field ligand, the electrons occupy $t_{2g}$ orbitals singly before pairing.
$t_{2g}^3 e_g^0$.
Number of unpaired electrons (n) = 3.
$\mu_s = \sqrt{3(3+2)} = \sqrt{3 \times 5} = \sqrt{15} \approx 3.87\operatorname{BM}$. This is not 4.90 BM.
3. [Mn(CN)\(_6\)]\(^{3-}\)
Oxidation state of Mn: Let it be $x$. $x + 6(-1) = -3 \implies x - 6 = -3 \implies x = +3$.
Manganese (Mn) has atomic number 25. Electronic configuration: [Ar] 3d\(^5\) 4s\(^2\).
Mn\(^{3+}\) electronic configuration: [Ar] 3d\(^4\).
CN\(^-\) (cyanide) is a very strong field ligand.
In the presence of a strong field ligand, the 3d\(^4\) electrons pair up.
$t_{2g}^4 e_g^0$. (This configuration is for low spin complex, one electron pairs up in $t_{2g}$).
Number of unpaired electrons (n) = 2.
$\mu_s = \sqrt{2(2+2)} = \sqrt{2 \times 4} = \sqrt{8} \approx 2.83\operatorname{BM}$. This is not 4.90 BM.
4. [MnCl\(_6\)]\(^{3-}\)
Oxidation state of Mn: Let it be $x$. $x + 6(-1) = -3 \implies x - 6 = -3 \implies x = +3$.
Manganese (Mn) has atomic number 25. Electronic configuration: [Ar] 3d\(^5\) 4s\(^2\).
Mn\(^{3+}\) electronic configuration: [Ar] 3d\(^4\). Cl\(^-\) (chloride) is a weak field ligand.
In the presence of a weak field ligand, the 3d\(^4\) electrons will follow Hund's rule and occupy orbitals singly as much as possible before pairing. This forms a high spin complex. $t_{2g}^3 e_g^1$.
Number of unpaired electrons (n) = 4.
$\mu_s = \sqrt{4(4+2)} = \sqrt{4 \times 6} = \sqrt{24} \approx 4.898\operatorname{BM}$. This matches 4.90 BM.
Step 3: Conclude the complex ion.
The complex ion [MnCl\(_6\)]\(^{3-}\) has 4 unpaired electrons and thus a spin-only magnetic moment of approximately 4.90 BM.
The final answer is $\boxed{[MnCl}_6]^{3-}}$.
List-I (Complex) | List-II (Isomerism) |
---|---|
A) [Co(NH3)5Br]SO4 | V) Ionization |
B) [Co(en)3]3+ | I) Optical |
C) [Co(NH3)5(NO2)]2+ | II) Linkage |
D) [Co(NH3)3Cl3] | III) Geometrical |
If \( \vec{u}, \vec{v}, \vec{w} \) are non-coplanar vectors and \( p, q \) are real numbers, then the equality:
\[ [3\vec{u} \quad p\vec{v} \quad p\vec{w}] - [p\vec{v} \quad \vec{w} \quad q\vec{u}] - [2\vec{w} \quad q\vec{v} \quad q\vec{u}] = 0 \]
holds for:
Statement-I: In the interval \( [0, 2\pi] \), the number of common solutions of the equations
\[ 2\sin^2\theta - \cos 2\theta = 0 \]
and
\[ 2\cos^2\theta - 3\sin\theta = 0 \]
is two.
Statement-II: The number of solutions of
\[ 2\cos^2\theta - 3\sin\theta = 0 \]
in \( [0, \pi] \) is two.
If \( A \) and \( B \) are acute angles satisfying
\[ 3\cos^2 A + 2\cos^2 B = 4 \]
and
\[ \frac{3 \sin A}{\sin B} = \frac{2 \cos B}{\cos A}, \]
Then \( A + 2B = \ ? \)